Траектория снаряда, выпущенного с вершины холма

Вот проблема:

Мальчик стоит на вершине холма, который равномерно спускается вниз под углом. ф . Под каким углом θ должен ли он бросать камень с горизонтали так, чтобы он имел наибольшую дальность?

Я понимаю, что тот же вопрос размещен здесь: https://physics.stackexchange.com/questions/24235/trajectory-of-projectile-thrown-downhill , но у меня есть несколько вопросов, на которые в этой теме не было ответа:

  1. Можно ли решить задачу без поворота системы координат? Если да, то как?
  2. Я попытался решить задачу с помощью повернутой системы координат, но не могу понять, как это закончить (см. работу, приведенную ниже).

Вот что у меня есть до сих пор:

  1. Зададим систему координат так, чтобы положительный Икс ось совпадает с нисходящим склоном холма. Это упрощает задачу, позволяя нам легко связать ф и θ , через отношение α "=" ф + θ .
  2. в 0 Икс "=" в 0 потому что α
  3. в 0 у "=" в 0 грех α
  4. а Икс "=" г потому что ( ф π 2 ) "=" г потому что ( ( π 2 ф ) ) "=" г потому что ( π 2 ф ) "=" г грех ф
  5. а у "=" г грех ( ф π 2 ) "=" г грех ( ( π 2 ф ) ) "=" г грех ( π 2 ф ) "=" г потому что ф
  6. в Икс "=" в 0 Икс + 0 т а Икс ( т ) г т "=" в 0 потому что α + 0 т ( г грех ф ) г т "=" в 0 потому что α + т ( г грех ф )
  7. Икс "=" Икс 0 + 0 т в Икс ( т ) г т "=" 0 т ( в 0 потому что α + т ( г грех ф ) ) г т "=" т ( в 0 потому что α ) + 1 2 т 2 ( г грех ф )
  8. в у "=" в 0 у + 0 т а у ( т ) г т "=" в 0 грех α + 0 т ( г потому что ф ) г т "=" в 0 грех α т ( г потому что ф )
  9. у "=" у 0 + 0 т в у ( т ) г т "=" 0 т ( в 0 грех α т ( г потому что ф ) ) г т "=" т ( в 0 грех α ) 1 2 т 2 ( г потому что ф )
  10. Чтобы найти время полета снаряда, найдем время пересечения его траектории с землей (в данном случае Икс ось), установив у "=" 0 и решение для т .
    у "=" т ( в 0 грех α ) 1 2 т 2 ( г потому что ф ) "=" 0
    в 0 грех α "=" 1 2 т ( г потому что ф )
    т "=" 2 в 0 грех α г потому что ф
  11. Замена т в уравнение для Икс дает нам расстояние, пройденное снарядом, как функцию углов α и ф .
    Икс "=" т ( в 0 потому что α ) + 1 2 т 2 ( г грех ф )
    Икс "=" ( 2 в 0 грех α г потому что ф ) ( в 0 потому что α ) + 1 2 ( 2 в 0 грех α г потому что ф ) 2 ( г грех ф )
    Икс "=" 2 в 0 2 г потому что ф ( грех α потому что α ) + 2 в 0 2 г потому что ф ( грех 2 α грех ф потому что ф )
    Икс "=" 2 в 0 2 г потому что ф ( грех α потому что α + грех 2 α загар ф )
  12. Я заметил, что решение в другом потоке исходит отсюда, дифференцируя Икс в отношении α , держа ф постоянная, что дает
    г Икс г α "=" 2 в 0 2 г потому что ф ( г г α ( 1 2 ( грех ( 2 α ) + грех 2 α загар ф ) )
    г Икс г α "=" 2 в 0 2 г потому что ф ( потому что ( 2 α ) + 2 грех α потому что α загар ф )
    г Икс г α "=" 2 в 0 2 г потому что ф ( потому что ( 2 α ) + грех ( 2 α ) загар ф )
    Это уравнение позволяет нам исследовать, как Икс меняется по отношению к α . Мы видим, что Икс увеличивается как α увеличивается до определенного момента, а затем уменьшается по мере α превышает это значение. Это означает, что график Икс имеет относительный максимум при значении α который обеспечивает максимальную дальность.
  13. Мы хотим найти значение α что приводит к максимальной дальности полета снаряда. Другими словами, мы должны определить значение α для которого график Икс имеет относительный максимум. Мы достигаем этого, устанавливая
    г Икс г α "=" 0 "=" 2 в 0 2 г потому что ф ( потому что ( 2 α ) + грех ( 2 α ) загар ф )
    Разделив каждую сторону на 2 в 0 2 г потому что ф производит
    потому что ( 2 α ) + грех ( 2 α ) загар ф "=" 0
    Вот где я теряюсь. Кажется, что это должно быть легкой частью, потому что осталось только решить приведенное выше уравнение для α , но я не знаю, как это сделать. Может ли кто-нибудь объяснить мне эту часть?

Дополнительно хотелось бы узнать, можно ли решить задачу без поворота системы координат. Первоначально я намеревался решить ее, используя стандартную прямоугольную систему координат, но увяз в некоторых уравнениях, которые, казалось, никуда не ведут. Спасибо за вашу помощь.

Прямоугольные координаты: у , как положительная высота, и Икс , как положительное расстояние, задаются как функции времени. Вы можете приравнять эти два через линию у "=" Икс загар ( ф ) . Тогда горизонтальный диапазон становится в 2 г ( грех ( 2 θ ) + загар ( ф ) потому что ( 2 θ ) + загар ( ф ) ) который после максимизации приводит вас куда-то близко к тому, где вы были.

Ответы (1)

Я не просмотрел всю вашу работу, но чтобы ответить на вопрос о том, как решить это уравнение, измените загар ( ф ) к грех ( ф ) потому что ( ф ) и возьми потому что ( ф ) как общий знаменатель. Вы получаете

потому что ( 2 α ) потому что ( ф ) + грех ( 2 α ) грех ( ф ) потому что ( ф ) "=" 0

Теперь примените идентификатор триггера потому что ( а б ) "=" потому что ( а ) потому что ( б ) + грех ( а ) грех ( б ) и ваше уравнение сводится к

потому что ( ф 2 α ) "=" 0
.